Pls answer 13 b ill mark u brainlist

Pls Answer 13 B Ill Mark U Brainlist

Answers

Answer 1

Answer:

7 students

Step-by-step explanation:

1+2+4=7students


Related Questions

The product of two numbers is 10000.If one number is 16 times the other numbers ,find the two numbers.​

Answers

▓▓▓▓▓▓▓▓▓▓▓▓▓▓▓▓▓▓

➯Question:

The product of two numbers is 10000. If one number is 16 times the other numbers ,find the two numbers.

Answer:

Both numbers are 25 and 400.

Step By Step Explanation:

Given that:

The product of two numbers is 10000.

One number is 16 times the other number.

To Find:

Both numbers?

Solution:

Let us consider that one number be n, in question it is stated that other number is 16 times the first number. Therefore, other number is 16n.

According to the Question :

[tex]\sf↦ Product ~of ~numbers = 10000\\\\↦ n × 16n = 10000\\\\↦ 16n² = 10000\\\\↦ n² = \sf \dfrac{10000}{16}\\\\↦ \sf n = \sqrt{\dfrac{10000}{16}}\\\\↦ \sf n = \sqrt{\dfrac{100\:\times\:100}{4\:\times\:4}}\\\\↦ \sf n = {\cancel{\dfrac{100}{4}}}\\\\➦\pmb{\underline{\boxed{\sf{\pink{n = 25}}}}}[/tex]

Hence,

1st number = n = 25

2nd number = 16n = 16 × 25 = 400

∴ Hence, both numbers are 25 and 400.

▓▓▓▓▓▓▓▓▓▓▓▓▓▓▓▓▓▓

Answer:

(25, 400) or (-25, - 400)

Step-by-step explanation:

Let the numbers be x and y.

We have:

y = 16xxy = 10000

Substitute y and solve for x:

x(16x) = 1000016x² = 10000x² = 10000/16x² = 625x = √625x = ± 25

Then y is:

y = ± 25*16 = ± 400

The numbers are:

25 and 400 or -25 and - 400

How many integers $n$ satisfy $-4n\ge 12$ and $n+5>2$??

Answers

-4n ≥ 12   ==>   n ≤ -3

n + 5 > 2   ==>   n > -3

But n cannot be both less than/equal to -3 AND greater than -3, so there are no integers n that satisfy these inequalities.

Answer: 0

Step-by-step explanation:

Solve the system of equations.
y = x2 + 3x - 4
y= 2x - 4

Answers

Answer:

The two solutions are

(0,-4) and (-1,-6)

Step-by-step explanation:

The ys have to be equal so you can equate the right hand side.

x^2 + 3x - 4 = 2x - 4         Add 4 to both sides

x^2 + 3x = 2x                    Subtract 2x from both sides

x^2 + 3x - 2x = 0              Combine

x^2 + x = 0                       Factor

x(x + 1) =0

Just to show you that these two points really are the solutions, you can see the graph below

x has 2 values

x + 1 = 0

x = - 1

or

x = 0

Now solve for y

y = 2x - 4

Let x = 0

y = - 4

Let x = - 1

y = 2x - 4

y = -2 - 4

y = - 6

The blue is y = x^2 + 3x - 4

The green line is y = 2x - 4

elizas backpack weights 18 7/8 pounds with her math book in it. without her math book, her backpack weighs 14 7/9 pounds

Answers

I'm assuming your trying to find the weight of the math book so here it is:

Answer:

4 5/36

Step-by-step explanation:

You'd have to do 18 7/8 - 14 7/9 but you can't subtract them yet because they don't have common denominators. So, you have to multiply the 7/8 by 9 and the 7/9 by 8. Then you would get 63/72 and 56/72. Then you put that into the equation and get, 18 63/72 - 14 53/72. If you subtract those, you get the answer, 4 10/72. Which can be simplified as 4 5/36.

Find the distance between the two points.
(5,5)
(2.2)
✓ [?]
Enter the number that
goes beneath the
radical symbol.

Answers

Answer:

[tex]{18}[/tex]

Step-by-step explanation:

[tex] \sqrt{(2 - 5) {}^{2} + ( 2 - 5) {}^{2} } [/tex]

[tex] = \sqrt{( - 3) {}^{2} + ( - 3) {}^{2} } [/tex]

[tex] = \sqrt{9 + 9} [/tex]

[tex] = \sqrt{18}[/tex]

Answered by GAUTHMATH

Please help! Geometry 9th grade

Answers

Answer:

69.3

Step-by-step explanation:

if you put it into 2D, you get a right triangle with leg lengths 62 (vertical height) and 31 (1/2 the base's length). Using Pythagorean's Theorem you get sqrt((62^2)+(31^2))= 69.318. Rounding to the nearest tenth you get 69.3

Find the measures of

Answers

Answer:

Step-by-step explanation:

Measure of an inscribed angle intercepted by an arc is half of the measure of the arc.

From the picture attached,

m(∠A) = [tex]\frac{1}{2}m(\text{arc BD})[/tex]

           = [tex]\frac{1}{2}[m(\text{BC})+m(\text{CD}][/tex]

           = [tex]\frac{1}{2}[55^{\circ}+145^{\circ}][/tex]

           = 100°

m(∠C) = [tex]\frac{1}{2}[(360^{\circ})-m(\text{arc BCD})][/tex]

           = [tex]\frac{1}{2}(360^{\circ}-200^{\circ})[/tex]

           = 80°

m(∠B) + m(∠D) = 180° [ABCD is cyclic quadrilateral]

115° + m(∠D) = 180°

m(∠D) = 65°

m(arc AC) = 2[m(∠D)]

m(arc AB) + m(arc BC) = 2(65°) [Since, m(arc AC) = m(arc AB) + m(arc BC)]

m(arc AB) + 55° = 130°

m(arc AB) = 75°

m(arc ADC) = 2(m∠B)

m(arc AD) + m(arc DC) = 2(115°)

m(arc AD) + 145° = 230°

m(arc AD) = 85°

Ashish is 175 cm tall his sister Annu is 8% shorter than him what is Annu's height

pls help​

Answers

Answer:

8% out of 175 = ( 8 ÷ 100 ) × 175 = 14

175 cm - 14 = 161 cm

...............

[tex]\sqrt[3]{x+1} =2x+2[/tex]

Answers

the answer is down below

How do I do question 2?

Answers

Answer:

518

Step-by-step explanation:

solution:

=8^2^×1^+1+6^2^×1^-1

=8^3+6^1

=8^3+6

=8×8×8+6

=512+6

=518

Fill in the table to explore the graph.
Miles Walked by Alonso
" Time (hours) 0.5 1.0 1.5 2.0 2.5 3.0 3.5 4.0
Miles Walked
6
V
16
12
Miles
8
4
X
1
4
3
Time (hours)
time

Answers

Answer:

Step-by-step explanation:

so you have divide  ok by my  and divide 5x1

What system of linear inequalities is shown in the graph?
Enter your answers in the boxes.

Answers

Answer:

y ≤ (-2)x+2

y < (1/3)x + 2

Step-by-step explanation:

First, we'll want to figure out the lines that are intersecting.

To find the equation of a line given two points (of form y=mx+b), we can first find the slope (m). This is equal to (y₂-y₁)/(x₂-x₁) for points (x₁,y₁) and (x₂,y₂). Then, we can plug a point in and solve for b.

For the line that has points at (1,0) and (2, -2), our slope is

(y₂-y₁)/(x₂-x₁) = (-2-0)/(2-1)

= (-2)/1

= -2

Our equation is therefore y= (-3/2)x+b. Plugging a point like (2,-2) in, we get

-2 = (-2)(2) + b

-2 = -4 + b

add 4 to both sides to isolate b

b =2

Our equation is thus y = (-2)x+2

For the other line, with points at (-3,1) and (0,2), our slope is (2-1)/(0-(-3)) = 1/3

Then, our equation is y=(1/3)x+b

plug (-3,1) in

1 = (1/3)(-3) + b

1 = -1 + b

add 1 to both sides to isolate the b

b =2

Our equation is thus y=(1/3)x + 2

Our equations are y=(1/3)x + 2 and y = (-2)x+2. In the graph shown, the shaded area is under both the lines (it is also to the left of them, not to the right or on top). Therefore, we can say that the shaded area is less than the line.

For y = (-2)x+2, the line is not dotted, so the line is included in the region. This means that our inequality is y ≤ (-2)x+2 (note the ≤ rather than simply the < sign)

For y=(1/3)x + 2, the line is dotted, so it is not included in the region. This means that our inequality is y < (1/3)x + 2

SOMEONE PLEASE HELP ME OUT THIS IS DUE In 20 MINUTES (PICTURE)

Answers

9ths answer. 113.112 ~ 113.1

On a recent math test. Samantha scored 3 points for each of the 18 multiple choice que non se
answered correctly and 5 points for each of the 6 short response questions she answered correctly
She scored an additional 4 points for the bonus question. Write an expression for her total score on
the test and then find her score.

Answers

The required expression will therefore be:

(3×18) + (5×6) + 4

Her total score is 88points :

From the question given, we are told that Samantha scored 3 points for each of 18multiple questions his means that the total points made here is expressed as:

(3 × 18) ...1

If she got 5 points for each of the 6 short responsmultiple choice questione questions she answered

correctly, the total point made will be expressed as;

(5×6) ... 2

Total score for an additional 4 points for the bonus question will be "4"

Expression for her total score will be the sum of all the expression above as thus:

(3×18) + (5×6) + 4

Taking the sum, her score will be:

= (3×18) + (5×6) + 4

= 54 + 30 + 4

= 84 + 4

= 88 points

The required expression will therefore be:

(3×18) + (5×6) + 4

Her total score is 88points

Learn more about sum here: brainly.com/question/16397188

how to change mixed fraction into improper fraction and improper to proper fraction easy way

Answers

To convert a mixed fraction to an improper fraction, follow these steps:

Step-by-step explanation:

Multiply the whole number part by the fraction's denominator.Add that to the numerator.Then write the result on top of the denominator

Hope this helps you ❤️

MaRk mE aS braiNliest ❤️

Answer with Step-by-step explanation:

To change the mixed fraction into improper fraction you multiply the denominator with the whole and add to numerator.

For example:  4 1/5 = (5*4)+1/5=21/5

To change improper into proper mixed fraction,

Divide the numerator by denominator and write the remainder on numerator

For example: 7/2= 3 1/2

. If a = 120°, find the measure of angles b, c and d.
Explain your reasoning.

Answers

∡b is a vertical angle to ∡a and therefore a twin, and we know that ∡a = 120°, thus since ∡b = ∡a, ∡b = 120° as well.

a full circle has a total of 360°, ∡d and ∡c are vertical angles, namely two angles across from each other at a junction, and therefore ∡d = ∡c.

∡a + ∡b is 120° + 120° thus 240°, since a circle has a total of 360°, 360 - 240 = 120, so the other two angles pick up that slack and divide it among each other evenly, 120/2 = 60, so ∡d = ∡c = 60°.

Find slopes of the given points .


[tex]\Large\star\tt {(5,6)\atop (3,-2)}[/tex]
[tex]\Large\star\tt {(-3,4)\atop (4,-7)}[/tex]


Spam free answers required .

Well explained answer will be marked as brainliest .​

Answers

Answer:

Use the slope formula:

m = (y₂ - y₁)/(x₂ - x₁)

The first one:

m = (-2 - 6)/(3 - 5) = -8/-2 = 4

The second one:

m = (-7 - 4)/(4 - (-3)) = -11/7

Answer:

Step-by-step explanation:

1)  (x₁,y₁) = (5 , 6)   &  (x₂,y₂) = (3 , -2)

[tex]Slope = \dfrac{y_{2}-y_{1}}{x_{2}-x_{1}}\\\\=\dfrac{-2-6}{3-5}\\\\=\dfrac{-8}{-2}\\\\=4[/tex]

2) (x₁,y₁) = (-3, 4)   &  (x₂,y₂) = (4 , -7)

[tex]Slope= \dfrac{-7-4}{4-[-3]}\\[/tex]

           [tex]= \dfrac{-11}{4+3}\\\\=\dfrac{-11}{7}[/tex]

What is the area of trapezoid DEFG?
PLEASE SO THIS ASAP!!!
MUST SHOW WORK

Answers

Answer:

Area = 35 cm^ 2

Step-by-step explanation:

Area = 1/2(a+b) x h

1/2 (5+9) x 5

= 35 cm^ 2

Answered by G a u t h m a t h

Answer:

A = 35  cm²

Step-by-step explanation:

The area (A) of a trapezoid is calculated as

A = [tex]\frac{1}{2}[/tex] h (b₁ + b₂)

where h is the perpendicular height between the bases and b₁, b₂ are the bases.

Here h = 5 , b₁ = 9 and b₂ = 5 , then

A = [tex]\frac{1}{2}[/tex] × 5 × (9 + 5) = 2.5 × 14 = 35 cm²

If f(x) = x2 + 1, what is the ordered pair for x =
-4.?

Answers

Answer:

(-4,17)

Step-by-step explanation:

y = f(x)

f(-4) = (-4)^2+1 = 17

y-coordinate = 17

Answer:

D). (-4, 17)

Step-by-step explanation:

Plug in -4 for x.

[tex]f(-4)=(-4)^2}+1[/tex]

Solve.

[tex]f(-4)=16+1[/tex]

[tex]f(-4)=17[/tex]

We already know that the x-coordinate is -4. (-4, y)

f(x) stands for y, so y=17.

(-4, 17)

I hope this helps!

How many tacos could you buy with your money? On Tuesday you can buy two tacos and you get one free.

If you had $20 dollars would you have enough money to buy a taco for 37 students on a Tuesday?

Answers

Answer:

due to not much information given and the way you ask the question I would definitely say no

Answer:

Yes you would have enough!

Step-by-step explanation:

Simply multiply .50 by 37 and that gives you 18.5!

i need help now! Mr. Morris is going to save money and replace his sailboat's mainsail himself. He must determine the area of the mainsail in order to buy the correct amount of material. Calculate the area of the parallelogram to determine how much material should be purchased. Be sure to explain how to decompose this shape into rectangles and triangles. Describe their dimensions and show your work.

Answers

Answer:

i cant see a parrellogram!

please post a picture

My hw says, "suppose E has a coordinate of -1 And EG= 7" what is the possible coordinate of G?​

Answers

Answer:

-8,6

Step-by-step explanation:

from what i can tell this is a 1D problem. |E-G|=7, and E=-1, so -1-G=+-7. G=-8 or 6

Answer: Wouldn’t the possible coordinate of G be 8?

Step-by-step explanation:

If E is -1 and we know that EG combined is 7, we can deduce that you have to have an 8 to get 7 because 8 + -1 = 7 or E + G = EG

simplify the monomial
11x^2 -3x^2

Answers

[tex]\sf11 {x}^{2} - 3 {x}^{2} \\ [/tex]

They are like terms so you can just easily subtract them directly.

[tex]\sf11 {x}^{2} - 3 {x}^{2} \\ = \boxed{\bf8x {}^{2}} [/tex]

Hope it helps.

RainbowSalt2222

[tex]\\ \sf\longmapsto 11x^2-3x^2[/tex]

Subtract the coefficients

[tex]\\ \sf\longmapsto (11-3)x^2[/tex]

[tex]\\ \sf\longmapsto 8x^2[/tex]

Which of the following numbers is between 0.08 and 0.4?
(A) 0.019
(B) 0.009
(C) 0.109
(D) 0.91
(E) 0.409

Answers

Answer:

C

Step-by-step explanation:

the numbers between 0.08 and 0.4 are:

0.09, 0.10, 0.11, 0.12, 0.13, 0.14, 0.15, .16, 0.17, 0.18, 0.19, 0.20, 0.21, 0.22, 0.23, 0.24, 0.25, 0.26, 0.27, 0.28, 0.29, 0.30, 0.31, 0.32, 0.33, 0.34, 0.35, 0.36, 0.37, 0.38, 0.39

I bolded the number closest to your answer choices, hope this helps!

Seventy-two books were to be divided up among 18 children. Which of the following
expressions is equivalent to this fraction?
a. 8/3 divided by 6/9
b. 8/3 times 2/3
c. 2/9 divided by 8/9
d. 2 times 8/2

Answers

Answer:

a

Step-by-step explanation:

cause if 72 book were divided by 18 kids they would hav all gotten 4 books each

so a is the answer cause it gives youthe answer 4

Answer: a. 8/3 divided by 6/9

Step-by-step explanation: I got this question on my test

HELLLPPPPPPPPP!!!!!!

Answers

ANSWER:

9) If the season is summer, then it also does not snow.

10) If you live in Salem, then you also live in Oregon.

(I’m not sure if I formatted these correctly in sentences, but hopefully you can see the pattern!)

What is the equation of the following graph in vertex form?
Y=(X-3)^2-1
Y=(X+3)^2-1
Y=(X-4)^2-2
Y=(X-4)^2+8

Answers

Answer:

Hello,

answer B

Step-by-step explanation:

Vertex is (-3,-1)

The equation of the parabola should be:

y=k*(x+3)²-1

(0,8) is a point of the parabola:

8=k*(0+3)²-1

8+1=9k

k=1

Equation is y=(x+3)²-1

Answer B

Please help!!!!!!!!!!​

Answers

Answer:

b) 7 (1, 2,3,5,7,11,13)

c) 0

d) 5

e) 8

A shopkeeper fixed the marked price of his radio to make a profit of 30 %. Allowing 15 % discount on the marked price, the radio was sold. What percent profit did he make? ​

Answers

Answer:

10.5 %

Step-by-step explanation:

Here .

let the Cost Price be 'x'

the..

by the question..

the marked price was fixed to make 30% profit

then..

the marked price will be

x+ 30% of x

= x + 30/100 × x

=x+ 0.3x

=1.3x

soo .. the marked price is 1.3x

now.. after allowing 15% discount..

the selling price will be

1.3x - 15% of 1.3x

= 1.3x - 0.195x

= 1.105x

thank you...

plzz mark me brainliest now

now..

profit made = 1.105x-x

= 0.105x

soo .

in percentage..

profit percent = profit/cp × 100%

= 0.105x/x × 100%

= 10.5 %

A number added by 7 is the same as that
number times four minus eleven.
Translate into an equation

Answers

Answer:

[tex]x+7=4x-11[/tex]

Step-by-step explanation:

Hi there!

Let [tex]x[/tex] represent "the number".

A number added by 7 is the same as that number times four minus eleven.

A number added by 7 ⇒ [tex]x+7[/tex]

Is the same as ⇒ [tex]x+7=[/tex]

The number times four ⇒ [tex]x+7=4x[/tex]

Minus eleven ⇒ [tex]x+7=4x-11[/tex]

I hope this helps!

Other Questions
If we aim a radio telescope at a distant spiral arm of the Milky Way Galaxy, we will probably observe a 21-cm line. If we point a large optical telescope at this same region, we will probably not be able to detect the neutral hydrogen that gives rise to the 21-cm radio signal. Why not importance of scientific discoveries Which of the following is NOT satirical? A converging lens of focal length 20cm, forms a real image twice the size of the object. Calculate: i) the power of the lens; ii) the position of the image. Find the equation of the midline of the function y = 2 sin(14x) 3.A) y = 3B) y = 3C) y = 2D) y = 14 Which of the following choices is equivalent to the equation below?5(2x1) = 5(5x14) A 2x 1 = 5x 14 B 5(2x 1) = 5x 14 C 2x 1 = 5 D None of these choices are correct. If a life insurance policy is purchased by someone who has noinsurable interest in the insured, it is considered An AM radio transmitter broadcasts 50.0 kW of power uniformly in all directions. I live 10 km from this station. What is the maximum strength of Electric Field in my house What does the conjugation of a word in a dictionary entry tell you about the word? what the various word endings are how it can be divided it into syllables how to pronounce the word the different synonyms of the word Which shows the correct substitution of the values a, b, and c from the equation -2 = -x + x2 4 into the quadraticformula?Quadratic formula: x =-bb2-4ac2 aOx=-(-1){V - 1)2 - 4(1)(-4)2(1)O x=-11/12-46- 1)( - 4)2(-1)O x= -13V (1)? - 4( - 1)(-2)2(-1)O x=-(-1)+7(-1)2 - 4(1)(-2)2(1) Prepare a profile of Sachin Ramesh Tendulkar Date of Birth : 24th April 1974 Place of Birth : Mumbai, Maharashtra Career : Cricketer, Captain of Indian Cricket Team. Famous as : One of the greatest cricketers of all times. Uniqueness : Only player to have scored a double century in a one day International, only player to complete more than 30,000 runs in International Cricket: How does cancer harm the body? What changes in cells allow cancer to develop? Height is 2ft. More than its width. If the total area is 99ft squared , determine the dimensions of the painting. 10. A recipe for punch calls for 2 cups of fruit concentrateand 6 cups of water.How many cups of punch will the recipe make? Please I need help please!!!!I need the answer ASAP!!!!!!If you know the answer please tell me The teacher was unaware of the difference between suspension feeding and predation. The teacher thought that providing live copepods (2 mm long) and feeder fish (2 cm long) would satisfy the dietary needs of all of the organisms. Consequently, which two organisms would have been among the first to starve to death (assuming they lack photosynthetic endosymbionts) given the functions w(x) =2x+1 and vw(x) =4x+4x-5,find w(x) answer meeeee You classified organisms based on anatomical structure and development. Scientists also use DNA to classify organisms. Considering the basic process of evolution, why do you think its important to also consider DNA and gene patterns during the classification of organisms? C2H5MgBr + PCl3 ra cht g Please help explanation if possible